What are stars made of?

Answers

Answer 1

Answer: The churning nuclear forges at the centres of stars produce light and heat, creating stars.

Explanation:

Answer 2

Answer: Hyrdrogen and Helium.

Explanation:


Related Questions

The capacitor can withstand a peak voltage of 670 v. If the voltage source operates at the resonance frequency

Answers

The value of maximum voltage, when the capacitor can withstand a peak voltage of 670 v is 47.6 volts.

What is resonance frequency?

Resonance frequency is the natural frequency of an particle or object. At this frequency, the object tries to vibrate at higher amplitude. It can be calculated with the following formula:

[tex]f=\dfrac{1}{2\pi LC}[/tex]

Here, L is the induction and C is the capacitance.

Let for an L-R-C circuit, the value of resistance is 400 ohm, induction is 0.380 Haney and capacitance is 1.20×10⁻² μF.

Put the values in above formula,

[tex]f=\dfrac{1}{2\pi(0.380)(1.20\times10^{-8})}\\f=2356.8\rm\; Hz[/tex]

The capacitor can withstand a peak voltage of 670 v. Use the following formula to find the value of current as,

[tex]I=2\pi fC\times Vc\\I=2\pi (2356.8)(1.20\times10^{-8})\times 670\\I=0.119\rm\; A[/tex]

The value of resistance is 400 ohms. Thus, the voltage is,

[tex]V=IR\\V=(0.119)(400)\\V=47.6\rm\; V[/tex]

Thus, the value of maximum voltage, when the capacitor can withstand a peak voltage of 670 v is 47.6 volts.

Learn more about the resonance frequency here:

https://brainly.com/question/3292140

#SPJ4

Both a transverse wave and a longitudinal wave have.

Answers

Answer:

Both waves are mechanical waves

What are voltage and current in electricity?

Answers

Answer:

Voltage, electric potential difference, electric pressure or electric tension is the difference in electric potential between two points, which is defined as the work needed per unit of charge to move a test charge between the two points.

Comets that come from the oort cloud have orbits that:
a. decay over time, bringing them closer to the sun each year.
b. have random tilts and orbits, sharing little with each other.
c. have very short orbits for their size and distance from the sun.
d. are evenly distributed between retrograde and prograde.

Answers

Comets that come from the Oort cloud have orbits that decay over time, bringing them closer to the sun each year. Option A is correct.

What are comets?

Comets are frozen leftovers from the formation of the solar system composed of dust, rock, and ice.

They range from a few miles to tens of miles wide, but as they orbit closer to the Sun, they heat up and spew gases and dust into a glowing head that can be larger than a plane.

Comets that come from the Oort cloud have orbits that decay over time, bringing them closer to the sun each year.

Hence,option A is correct.

To learn more about comet refer:

https://brainly.com/question/12443607

#SPJ1

Answer:

B) have random tilts and orbits, sharing little with each other

Explanation:

Hope this helps, have a great rest of your day/ restful night!

During a car accident, a 125 kg driver is moving at 31 m/s and in 1.5 seconds is brought to rest by an inflating air bag. what is the magnitude of the change in momentum of the driver? 4.0 kg • meters per second 21 kg • meters per second 47 kg • meters per second 3900 kg • meters per second

Answers

The magnitude of the change in momentum of the driver is 3900kgm/s.

Hence option D) 3900 kg • meters per second is the correct answer.

What is Momentum?

Momentum is simply the product of the mass of an object and its velocity.

Its is expressed as;

P = m × v

Where m is the mass of the object and v is its velocity.

Given the data in the question;

Mass of the driver m = 125kgVelocity v = 31m/sTime elapsed t = 1.5s

We substitute the given values into the expression above to determine the initial momentum of the driver before he was brought to rest.

P initial = m × v

P initial = 125kg × 31m/s

P initial = 3875 kgm/s ≈ 3900kgm/s

Now, as the driver is brought to rest, final velocity becomes zero

P final = 125kg × 0

P final = 0

This means that the change in momentum of the driver is his initial momentum which is 3900kgm/s.

Therefore, the magnitude of the change in momentum of the driver is 3900kgm/s.

Hence option D) 3900 kg • meters per second is the correct answer.

Learn more about momentum here: brainly.com/question/14952164

#SPJ4

Answer:

D for edge

Explanation:

GO GO GO TRUST ME

calculate the resistance of the 1000 W microwave oven that gets plugged into a 110 v outlet

Answers

The resistance of the 1000 W microwave oven that gets plugged into a 110 v outlet will be 10 ohms.

What is resistance?

Resistance is a type of opposition force due to which the flow of current is reduced in the material or wire. Resistance is the enemy of the flow of current.

The given data in the problem is;

P is the power= 1000 W

V is the voltage= 110 V

I is the current

R is the resistance

The resistance is found by the formula;

[tex]\rm P = \frac{V^2}{R} \\\\ \rm R = \frac{V^2}{P} \\\\ R = \frac{(100)^2}{10000} \\\\ R= 10\ ohm[/tex]

Hence, the resistance value will be 10 ohms.

To learn more about the resistance, refer to the link;

https://brainly.com/question/20708652

#SPJ1

0.318 cm. What is the ideal mechanical advantage?

Answers

The ideal mechanical advantage will be 8.017. The ideal mechanical advantage value may be grater than the actual value of mechanical advantage.

What is mechanical advantage ?

Mechanical advantage is a measure of the ratio of output force to input force in a system, it is used to obtained efficiency of  forces in levers and pulley.

The ideal mechanical advantage is found as;

[tex]\rm IMA = \frac{d \pi}{l} \\\\ \rm IMA = \frac{0.812 \times \pi}{0.318} \\\\ IMA=8.017[/tex]

The ideal mechanical advantage will be 8.017.

To learn more about the mechanical advantage, refer to the link;

https://brainly.com/question/7638820

#SPJ1

A mouse eats 20 kJ of energy. The mouse uses 8 kJ of energy moving around. What would most of the leftover energy be turned into? ant is the leftover energy useful or wasted?

Answers

Answer:

to be turned into glycogen not wasted

Explanation:

Since the body has a lot of energy left it will turn that energy into glycogen which is a storage form of glucose which will when neccassary will be converted into glucose from the horomone glucagon and be given to the body to be used as energy so the energy is never wasted

physics 13

physics geniuses where r u need help!!!
----------------------


pls also check other questions!

Answers

Answer:

Microphone

Explanation:

Microphone is the device which converts sound into electrical energy by electromagnetic induction

an example of inertia is when your body flies forward when you slam on the breaks.

a,true
b,false

help :​

Answers

Answer:

A True ( I Think Its True I'm Not Intirely sure)

Explanation:

Answer:

true

Explanation:

True.....when you apply the brakes to slow down, your body's inertia wants ot keep moving forward

How much work done if an object is moving to the right and a force of 75. 0n is applied at 125 degrees to the motion while the object travels 7. 50m

Answers

Answer:

-322.64 J

Explanation:

75 N  * cos 125     *    7.5   =  -322.64 J

The voltage across two points in an electric circuit is
876 mV. A power source does 1 901 kJ of work
when moving a certain amount of charge between
these two points.
Determine the amount of charge that moved between
between the two points.

Answers

Answer:

V=w/q V= 1.2/0.4* 10 ^-3 V = 3 * 10 ^ 3 V = 3000 v

if its not correct let me know so I can figure it out

What question is maya most likely trying to answer? how does the type of material used in the core affect the strength of an electromagnet? how does the number of loops of wire affect the strength of an electromagnet? how does the speed at which an iron bar is removed from wire loops affect the strength of an electromagnet? how does the thickness of the wire affect the strength of an electromagnet?

Answers

The question is maya most likely trying to answer is, how does the thickness of the wire affect the strength of an electromagnet?

What is effect of thickness of wire on strength of electromagnet?

From the experimental set-up by maya, we can determine the effect of thickness of wire on strength of electromagnet.

R ∝ 1/A

where;

R is resistance of the wiresA is area of the wires (from thickness or radius of the wire)

As the thickness of the wire increases, the area of the wire increases and the resistance of the wire will decrease. As the resistance of the wire decreases, the current flowing in the wire increases as well.

Thus, the increase in the thickness of a wires increases, the strength of an electromagnet.

Learn more about strength of an electromagnet here: https://brainly.com/question/2331156

#SPJ4

Answer:

A

Explanation:

What portion of the electromagnetic spectrum is used for mass spectrometry

Answers

The energy of the electromagnetic spectrum is not used in mass spectrography to make measurements.

What is mass spectrometry?

In physics and chemistry, mass spectrometry refers to statistical analytical techniques that allow scientists to determine the mass distribution of various types of molecules based on their mass on a substance.

The energy of the electromagnetic spectrum is not used in mass spectrography to make measurements. The process of mass spectrometry is primarily based on the interaction of molecules with a beam of electrons (rather than photons) and the subsequent measurement.

Hence the energy of the electromagnetic spectrum is not used in mass spectrography to make measurements.

To know more about Mass spectrometry follow

https://brainly.com/question/17368088

#SPJ4

1- The wave length is the distance between the middle of a crest to the middle of the next trough

true
or
false

Answers

Answer:

False

Explanation:

Middle of crest to middle of NEXT CREST is wavelength

Explain How did Aristotle's inability to detect parallax lead him to propose a geocentric model of the solar system?​

Answers

Answer:

Aristotle was blind, therefore could not see change in positions. Aristotle could see no detectable change in the positions of the stars, thought Earth was immovable.

Explanation:

A wire is run through a large piece of paper, forming a perpendicular angle with the paper. A bunch of magentically senstive iron shavings are spread across the paper and a current is run through the wire. Which of the following would be the most accurate description of what would happen?


A. The filings would shape themselves into a square shape around the wire, each corner of the square perpendicular to the central wire

B. The filings would form parallel lines across the paper, each one forming a perpendicular angle with the central wire

C. The filings would shape themselves into a circular shape around the wire, the direction rotating based on the direction of the flow of current

D. The filings would all be attracted magnetically towards the wire and will all be pulled into a central mass in close proximity to the wire and will leave the outer part of the sheet of paper bare

Answers

Answer:

Option C

Explanation:

As they are iron shavings they are magnetic i.e they can be attracted by magnet .So when electric current is passed through the wire they would form an electromagnetic field .And the shape is circular

Ip three resistors, r1r1r_1 = 11 ωω , r2r2r_2 = 68 ωω , and r3=rr3=r, are connected in series with a 24. 0 vv battery. The total current flowing through the battery is 0. 20 aa

Answers

The three resistances are connected in series, their equivalent is equal to the addition of all the resistances. The unknown value of resistance is 1.74 ohm.

What is resistance?

The resistance is the opposition offered to the flow of current.

Given the value of resistance R₁ is 1 ohm, R₂ is 68 ohm and R₃ is R.

The total current flowing through the battery of voltage 24 V is 0.20 Amperes.

From the ohms law, The equivalent resistance is

[tex]R_{eq}[/tex] = Voltage / Current

The equivalent resistance is

[tex]R_{eq}[/tex] = R₁+ R₂+R₃

24 / 0.20 = 1+68+R

R = 1.74 ohm

The unknown value of resistance is 1.74 ohm.

Learn more about resistance.

https://brainly.com/question/11431009

#SPJ4

As altitude increases, what happens to air pressure?

it fluctuates.
it increases.
it decreases.
it stays the same.

Answers

The air pressure decreases

As altitude increases, air pressure decreases. hence option C is correct.

What is Atmospheric pressure ?

It is pressure exerted by the weight of the atmosphere at sea level. its value is 101,325 Pascal which we call it as 1 atmospheric pressure expressed as 1 atm. i.e. 1 atm = 101,325 N/m².

Like water has pressure when we go long inside the water, our atmosphere has also pressure from top to surface of the earth.

There are different layers of atmosphere from bottom to top which are Troposphere, Stratosphere, Mesosphere, Thermosphere and Exosphere.

Each layer has its own range and weight.

When we are on the surface of the earth, all layers of atmosphere will apply pressure. but when we increase altitude(height), the number layers reduces and its weight too.

Hence as altitude increases, air pressure decreases.

To know more about pressure, click :

https://brainly.com/question/12497098

#SPJ5

* if air is blown with a speed of 15 m/sec across the top of one side of a u-tube containing water, what will be the difference between the water levels on the two sides?

Answers

if air is blown with a speed of 15 m/sec across the top of one side of a u-tube containing water, the difference between the water levels on the two sides is 1.3 cm.

What is density?

The density is the ratio of the mass and volume of the particular object or liquid.

Applying the Bernoulli's equation across the ends of the tube A and B, we have

[tex]P_{A} = P_{B} - \dfrac{1}{2} \rho Av^{2}[/tex]

Substituting ρ = 1.2 , v = 15 and [tex]P_{B }=P_{0 }[/tex]

[tex]P_{A}= P_{B} -135[/tex]

In the left limb, [tex]P_{A}= P_{C}[/tex]

In the right limb, [tex]P_{D}= P_{o} + \rho g h[/tex]

The pressure at C and D are same.

[tex]P_{D}= P_{C}[/tex]

Substituting from the above equations , we have,

h = -1.3 cm

The height to the left limb is greater.

The difference between the water levels on the two sides is 1.3 cm.

Learn more about density.

https://brainly.com/question/15164682

#SPJ4

An electron and a 0.0500-kg bullet each have a velocity of magnitude 510 m/s, accurate to within 0.0100%. Within what lower limit could we determine the position of each object along the direction of the velocity

Answers

The lower limit that could be used to determine the position of each object along the direction of the velocity is 1.136 x 10⁻³ m.

Uncertainty in the position of the electron

The uncertainty in the position of the electron is calculated as follows;

[tex]\Delta x = \frac{h}{4\pi m\Delta v}[/tex]

where;

h is Planck's constant = 6.63 x 10⁻³⁴ Jsm is mass of electronΔv uncertainty in velocity = (0.01 x 10⁻²) x 510 = 0.051 m/s

[tex]\Delta x = \frac{6.63 \times 10^{-34} }{4\pi \times 9.11 \times 10^{-31} \times 0.051} \\\\\Delta x = 1.136 \times 10^{-3} \ m[/tex]

Thus, the lower limit that could be used to determine the position of each object along the direction of the velocity is 1.136 x 10⁻³ m.

Learn more about uncertainty in position here: https://brainly.com/question/1970053

#SPJ1

Which types of electromagnetic waves have higher frequencies than the waves that make up ultraviolet light? Select two options. Radio waves infrared light microwaves gamma rays visible light X-rays.

Answers

Answer:

the answer is equal to the microwave gamma rays visible light x-rays.

Contrast series and parallel circuits in terms of what they look like and how they calculate Equivalent resistance

Answers

In a series circuit the current through each of the components is the same, and the voltage across the circuit is the sum of the voltages across each component.

In a parallel circuit, the voltage across each of the components is the same, and the total current is the sum of the currents through each component.

Let's see

For series connection

[tex]\\ \rm\Rrightarrow R_{eq}=R_1+R_2+R_3\dots[/tex]

For parallel connection

[tex]\\ \rm\Rrightarrow \dfrac{1}{R_{eq}}=\dfrac{1}{R_1}+\dfrac{1}{R_2}+\dfrac{1}{R_3}\dots[/tex]

What is an animal that lays eggs in nests and some of it's genuses lack the ability
hint: it is 5 letters long
another hint : _ _ r _ s

Answers

Answer:

I think is cookoo because i learn it from web

Explanation:

I found it in web so yea

The magnitude of the earth pulling on you is equivalent to your body weight.
O A. True
OB. False

Answers

Answer:

I think its O A. True.

A string of 5 (Five) 15 ohm Christmas tree lights are connected in parallel. One burns out, the rest will stay lit. Calculate the total resistance when one of the bulbs is out.

Pt 2.

Those five 15 ohm, series connected Christmas tree lights, calculate the total current in the circuit if they are connected to a 115 V source. What would be total power?

Pt. 3
Those five 15 ohm parallel connected Christmas tree lights. Calculate the total current in the circuit if they are connected to a 115 V source. Calculate total Power.

Please list formula and data etc etc

Answers

Answer:

r = 15 ohms    for a single bulb

1 / R = 1 / r  + 1 / r + 1 / r + 1 / r

R = r / 4 = 15 / 4 = 3.75 ohms       total resistance

part II

If bulbs are in series R = 5 * 15 ohm = 75 ohm total resistance

P = I V = V / R * V = V^2 / R

P = 115^2 / 75 = 176 watts

part III

If the bulbs are in parallel then      

R = 15 / 5 =  3 ohms for the circuit

I = 115 / 3 = 38.3 amps for circuit

P = 115^2 / 3 = 4400 watts

Check: P = I V = 38.3 * 115 = 4400 watts

An object has a velocity of 8 m/s and a kinetic energy of 480 J. What is the mass of the object? (Formula: ) 7. 5 kg 15 kg 60 kg 120 kg.

Answers

Answer:

15 kg

Explanation:

KE = 1/2 * m * v²   ⇒  m = (2*KE) / v²

m = (2*480) / 8²

m=15 kg

the period of a 400 hertz sound wave is ___ second

Answers

Answer:

0.0025 sec

Explanation:

Period = 1 / frequency = 1/400 = 0.0025 sec

0.0025 per second your welcome

The trait for flower color in a plant has red and white alleles. The red color is the dominant trait. What is the phenotypic ratio for a cross between plants with red (Rr) flowers and red (Rr) flowers?

4 red : 0 white
3 red : 1 white
2 red : 2 white
1 red : 1 white

Answers

Answer:

1.4 red : 0 white

Explanation:

Answer: A

Explanation:

In real number multiplication, if uv1=uv2 and u does not equal zero we can cancel out u and conclude that v1=v2. Does the same rule hold for the dot product?

Answers

The conclusion v₁=v₂ can not be made. Let u=(1,1,1),v₁=(2,3,-5),v₂=(4,-1,-3). The dot product of u.v₁=uv₂ is satisfied,but v₁≠v₂.The correct answer is c.

What is dot product?

The dot product, also known as the scalar product, is an algebraic operation that yields a single integer from two equal-length sequences of numbers.

The dot product of two vectors' Cartesian coordinates is commonly used in Euclidean geometry.

Because there may be more than the 1 vectors satisfying the expression.

[tex]\rm uv_1 = 1 \times 2 +1 \times 3 +1 \times(-5)=0[/tex]

[tex]\rm uv_2= 4-1-3 =0[/tex]

The v₁ is not found equal to the v₂.

The conclusion v₁=v₂ can not be made. Let u=(1,1,1),v₁=(2,3,-5),v₂=(4,-1,-3). The dot product of u.v₁=uv₂ is satisfied,but v₁≠v₂.The correct answer is c.

Hence, the correct answer is c.

To learn more about the dot product, refer to the link;

https://brainly.com/question/26550859

#SPJ4

Other Questions
ethanol is widely used in antibacterial hand sanitizer and wipes.explain Dr. Strauss has a bottle that contains 18 ounces of medicine. How many one-fourth ounce doses are in 18 ounces? How much energy is contained in a particle that has a mass of 1x10^-9 kg? Scarce resources and social rewards are distributed on the basis of ascribed statuses in a. a caste system. c. a class system. b. an open system. d. a closed system. Please help ASAP!!!Write an essay explaining why it is important to forgive. Who did the people of Japan turn toafter the Fujiwara lost power?A. priestsB. clansC. daimyos Natalia paid $38.95 for three medium-sized pizzas and a salad. if Natalia paid $11.98 for the salad, which equation can be used to find how much did each pizza cost?A. 3m + $11.98 = $38.95B. 4m + $11.98 = $38.95C. 3m + $38.95 = $11.98D. none of thesePLEASE HELP!! How did the ideas of the Scientific Revolution transform Europe? Two days ago, Susan was 18. Yesterday was her birthday and she is now 19. Next year, she will be 21. When is her birthday? Match the statement to the correct document.a. Declaration of Independenceb. Declaration of Sentiments1. We hold these truths to be self-evident, that all men were created equal...2. We hold these truths to be self-evident, that all men and women were created equal.- Determine the slope of the line that runs through points (-3,6) And (0,-8) using the slope formula. YT -- YTM= -------------- X2 X2 TASK 1: BUDGET AND NET WORTH Keep track of your income and expenses for one month. List all your assets and liabilities (debts owed) in a spreadsheet. Based on these documents, develop a monthly budget and a net worth statement. Find the value of x in the parallelogram below . Find the United States on the world map and Where is the equator? PLEASE HELPPP In order to determine the best fit undergraduate program, a high school senior researches the majors offered at the universities he is considering for admissions. The internet search reports that Arizona State University lists nine of their most popular majors and the percent of student population enrolled in the programs. Brand XYZ dog food costs $36 for 15 pounds. How much would 24 pounds of dog food cost? Study the following graph and answer the questions that follow: if a health care worker suspects a patients is being abused, why does she have to report it? What occurs as the atomic number of the elements in period to increases Which would be most appropriate to help the polar bears? international laws an international treaty the world wildlife fund the environmental protection agency